Skip to Main Content

PrepTest 78, Game 3, Question 12

Transcript

The first question of PT 78. Game number 3, says what could be an accurate list of a six antiques in order? And for this kind of question, the best strategy is to use our rules to eliminate answers that don't follow them. So remember the first rule said S can't go first. That's going to get rid of answer choice D because there S is first, the second rule says if H is before L, then M also has to be before L.

And so that is gonna get rid of answer choice A, where you see the H before L, but you see the M after L, the M is supposed to go before the L. Rule number 3, was that S has to come before both M and V. L times S is before M and V, that gets rid of answer choice E which has V before S. And then the last rule was that T has to come before H or V but not both. And that means that T has to come in between V and H.

Either it's gonna be V before T before H or H-T-V, that gives sort of answer choice B, because there you see T is at the end. It's not in between H and V. That's why C is our answer.

Read full transcript